matheraum.de
Raum für Mathematik
Offene Informations- und Nachhilfegemeinschaft

Für Schüler, Studenten, Lehrer, Mathematik-Interessierte.
Hallo Gast!einloggen | registrieren ]
Startseite · Forum · Wissen · Kurse · Mitglieder · Team · Impressum
Forenbaum
^ Forenbaum
Status Schulmathe
  Status Primarstufe
  Status Mathe Klassen 5-7
  Status Mathe Klassen 8-10
  Status Oberstufenmathe
    Status Schul-Analysis
    Status Lin. Algebra/Vektor
    Status Stochastik
    Status Abivorbereitung
  Status Mathe-Wettbewerbe
    Status Bundeswettb. Mathe
    Status Deutsche MO
    Status Internationale MO
    Status MO andere Länder
    Status Känguru
  Status Sonstiges

Gezeigt werden alle Foren bis zur Tiefe 2

Navigation
 Startseite...
 Neuerdings beta neu
 Forum...
 vorwissen...
 vorkurse...
 Werkzeuge...
 Nachhilfevermittlung beta...
 Online-Spiele beta
 Suchen
 Verein...
 Impressum
Das Projekt
Server und Internetanbindung werden durch Spenden finanziert.
Organisiert wird das Projekt von unserem Koordinatorenteam.
Hunderte Mitglieder helfen ehrenamtlich in unseren moderierten Foren.
Anbieter der Seite ist der gemeinnützige Verein "Vorhilfe.de e.V.".
Partnerseiten
Weitere Fächer:

Open Source FunktionenplotterFunkyPlot: Kostenloser und quelloffener Funktionenplotter für Linux und andere Betriebssysteme
StartseiteMatheForenUni-StochastikErwartungswert
Foren für weitere Studienfächer findest Du auf www.vorhilfe.de z.B. Astronomie • Medizin • Elektrotechnik • Maschinenbau • Bauingenieurwesen • Jura • Psychologie • Geowissenschaften
Forum "Uni-Stochastik" - Erwartungswert
Erwartungswert < Stochastik < Hochschule < Mathe < Vorhilfe
Ansicht: [ geschachtelt ] | ^ Forum "Uni-Stochastik"  | ^^ Alle Foren  | ^ Forenbaum  | Materialien

Erwartungswert: Aufgabe
Status: (Frage) beantwortet Status 
Datum: 16:39 So 09.01.2005
Autor: karin1982

Hallo!

Sietze mal wieder vor einen Stochastik-Aufgabe und komme nicht weiter:

Roulette-Spiel:
Man spielt über mehrere Runden und setzt jedesmal auf "rot", bis zum ersten Mal tatsächlich "rot" kommt
(WSK [mm] \bruch{18}{27}). [/mm]
Dann beendet man das Spiel.
Die Höhe des Einstzes beträgt in der 1. Runde 1 Euro, in der ggf. 2. Runde 2 Euro,
allg. in der n-ten Runde (falls man so lang spielen muss: [mm] 2^{n-1} [/mm] Euro.
Beim Roulette erh. man beim Setzen auf "rot" den doppeltren Einsatz zurück,
falls auch "rot" kommt, ansonsten verfällt der Einsatz.

a)
Was sind Verteilung & Erwartungswert des Gesamtgewinnes X, wenn man die Strategie bis zum Ende durchhält?
Wie lange muss man im Mittel spielen?
Welchen Betrag hat man im Mittel bis zur letzten Runde gesetzt?

b)
Wenn man über 1,1 Mrd. Euro verfügt, können Sie die Verdopplungs-Strategie höchstens bis zur 30. Runde durchstehen.
Wie sieht die Verteilung und der Erwartungswert des Gesamtgewinns X aus,
wenn das Spiel nach der 30. Runde abgebrochen wird?

Kann mir jemand weiterhelfen?
Wäre Euch sehr dankbar!
GuK
Karin

        
Bezug
Erwartungswert: Antwort
Status: (Antwort) fertig Status 
Datum: 18:49 So 09.01.2005
Autor: holy_diver_80

Hallo karin1982!

a) in der 1. Runde hast Du 1 Euro gesetzt und kannst 2 gewinnen. Der Reingewinn beträgt dann 1 Euro, fall Du gewinnst. in der zweiten Runde hättest Du 1 Euro verloren, 2 gesetzt und kannst 4 gewinnen. wenn Du gewinnst, machst Du also einen Euro Gewinn. Wenn Du Dir die geometrische Reihe für den Einsatz ansieht und mit dem möglichen Gewinn in dieser Runde vergleichst erhälts Du allgemein.
Einsatz in der k-ten Runde: [mm] \summe_{i=0}^{k-1} 2^{i} [/mm] = [mm] 2^{k}-1 [/mm]
Gewinn in der k-ten Runde: [mm] 2*2^{k-1} [/mm] = [mm] 2^{k} [/mm]
Differenz: 1
Falls Du gewinnst - und irgendwann tust Du das - gewinnst Du 1 Euro. Daher ist das auch der Erwartungswert für den Gewinn.

Sei nun X die Zufallsvariable, die mißt, in welcher Runde Du gewinnst. Diese ist geometrisch verteilt. D.h. es gilt: p(X=k) = [mm] (1-p)^{k-1} [/mm] * p, wobei p die Gewinnwahrscheinlichkeit ist, in unserem Fall ist p=18/37.
Wir suchen nun das kleinste n, für das gilt [mm] \summe_{k=1}^{n} [/mm] p(X=k) [mm] \ge [/mm] 1/2
Dieses n ist dan die wahrscheinliche Spieldauer. Nach Verwandlung der Summe in eine geometrische Reihe erhält man n=2. Man muß also in der Regel nur 2 Runden spielen, bis man Erfolg hat.
Da die Wahrscheinlichkeit inder 1. Runde zu gewinnen gleich 18/37 ist, und die in der 2. zu gewinnen gleich (19/37)*(18/37) ist und die Einsätze bis dahin 1 bzw 3 Euro betrugen, ist der durchschnittliche Einsatz bis zum Gewinn gleich
(18/37)*1 + (19/37)*(18/37)*3 =1,24 Euro

b) Wenn Du 30 Runden spielt und dann abbrichst, dann ist die Wahscheinlichkeit, immer zu verlieren gleich q=(19/37)^30, die Wahrscheinlichkeit einmal zu gewinnen entsprechend 1-q.
Bis zur 30. Runde hast Du 2^30-1 Euro ist Spiel gesteckt. Der Erwartungswert des Gewinns ist dann
(1-q)*1-q*(2^30-1)=-1.225646449
Zur Verteilung:
p(Gewinn = 1) = 1-q
p(Gewinn = -2^30+1) = q
p(sonst) = 0

Du siehst, wenn Du ewig spielen könnest, gewinnst Du sicher, wenn Du nur 30 Runden spielen darfst, dann verlierst Du wahrscheinlich.

Bezug
                
Bezug
Erwartungswert: Danke
Status: (Mitteilung) Reaktion unnötig Status 
Datum: 14:32 Do 13.01.2005
Autor: karin1982

Vielen Dank für Deine nette Hilfe!

GuK
Karin

Bezug
Ansicht: [ geschachtelt ] | ^ Forum "Uni-Stochastik"  | ^^ Alle Foren  | ^ Forenbaum  | Materialien


^ Seitenanfang ^
www.schulmatheforum.de
[ Startseite | Forum | Wissen | Kurse | Mitglieder | Team | Impressum ]